Difference between revisions of "1990 AJHSME Problems/Problem 2"

(Created page with '==Problem== Which digit of <math>.12345</math>, when changed to <math>9</math>, gives the largest number? <math>\text{(A)}\ 1 \qquad \text{(B)}\ 2 \qquad \text{(C)}\ 3 \qquad \…')
 
 
Line 13: Line 13:
 
{{AJHSME box|year=1990|num-b=1|num-a=3}}
 
{{AJHSME box|year=1990|num-b=1|num-a=3}}
 
[[Category:Introductory Algebra Problems]]
 
[[Category:Introductory Algebra Problems]]
 +
{{MAA Notice}}

Latest revision as of 00:04, 5 July 2013

Problem

Which digit of $.12345$, when changed to $9$, gives the largest number?

$\text{(A)}\ 1 \qquad \text{(B)}\ 2 \qquad \text{(C)}\ 3 \qquad \text{(D)}\ 4 \qquad \text{(E)}\ 5$

Solution

When dealing with positive decimals, the leftmost digits affect the change in value more. Thus, to get the largest number, we change the $1$ to a $9 \rightarrow \boxed{\text{A}}$.

See Also

1990 AJHSME (ProblemsAnswer KeyResources)
Preceded by
Problem 1
Followed by
Problem 3
1 2 3 4 5 6 7 8 9 10 11 12 13 14 15 16 17 18 19 20 21 22 23 24 25
All AJHSME/AMC 8 Problems and Solutions

The problems on this page are copyrighted by the Mathematical Association of America's American Mathematics Competitions. AMC logo.png